Can an indefinite integral be expressed as a definite integral with variable bounds?












2














If I have a function $f(t)$, and an indefinite integral of this function, $g(x) = int f(t), dt$, is there any way I can express $g(x)$ as a definite integral whose bounds depend on $x$?



I thought I saw somewhere that I could express it as
$$g(x) = int_a^x f(t), dt$$
but I don't know what $a$ is. Is this representation correct? Or is there a better way to express $g$?










share|cite|improve this question
























  • The second representation is correct. The first one doesn't represent $g$ as a function of $x$.
    – user23793
    3 hours ago
















2














If I have a function $f(t)$, and an indefinite integral of this function, $g(x) = int f(t), dt$, is there any way I can express $g(x)$ as a definite integral whose bounds depend on $x$?



I thought I saw somewhere that I could express it as
$$g(x) = int_a^x f(t), dt$$
but I don't know what $a$ is. Is this representation correct? Or is there a better way to express $g$?










share|cite|improve this question
























  • The second representation is correct. The first one doesn't represent $g$ as a function of $x$.
    – user23793
    3 hours ago














2












2








2







If I have a function $f(t)$, and an indefinite integral of this function, $g(x) = int f(t), dt$, is there any way I can express $g(x)$ as a definite integral whose bounds depend on $x$?



I thought I saw somewhere that I could express it as
$$g(x) = int_a^x f(t), dt$$
but I don't know what $a$ is. Is this representation correct? Or is there a better way to express $g$?










share|cite|improve this question















If I have a function $f(t)$, and an indefinite integral of this function, $g(x) = int f(t), dt$, is there any way I can express $g(x)$ as a definite integral whose bounds depend on $x$?



I thought I saw somewhere that I could express it as
$$g(x) = int_a^x f(t), dt$$
but I don't know what $a$ is. Is this representation correct? Or is there a better way to express $g$?







calculus definite-integrals indefinite-integrals






share|cite|improve this question















share|cite|improve this question













share|cite|improve this question




share|cite|improve this question








edited 3 hours ago









Ethan Bolker

41.2k547108




41.2k547108










asked 3 hours ago









clabe45

1295




1295












  • The second representation is correct. The first one doesn't represent $g$ as a function of $x$.
    – user23793
    3 hours ago


















  • The second representation is correct. The first one doesn't represent $g$ as a function of $x$.
    – user23793
    3 hours ago
















The second representation is correct. The first one doesn't represent $g$ as a function of $x$.
– user23793
3 hours ago




The second representation is correct. The first one doesn't represent $g$ as a function of $x$.
– user23793
3 hours ago










2 Answers
2






active

oldest

votes


















5














No, not always.



In fact, the definite integral with variable bounds can be and is strictly more restrictive than the indefinite integral, in the most general case where both can be taken as sensibly existing to their fullest extent that we can say one defines the same kind of function as the other.



While it is true that if we have two functions $F_1$ and $F_2$ defined by definite integrals of the same function with different bounds, i.e.



$$F_1(x) := int_{a_1}^{x} f(t) dt mathrm{and} F_2(x) := int_{a_2}^{x} f(t) dt$$



with $a_1 ne a_2$, then we will have that $F_1 - F_2$ is a constant function, the range of possible such constants may be limited. A simple counterexample is to take $f(x) := sin(x)$. Now the integral from any lower bound $a$ to $x$ is just $int_a^x sin(t) dt = cos(a) - cos(x)$, but here $cos(a)$ - the "constant" of our integration - can only range in $[-1, 1]$ (and the difference of any two such "constants" only within $[-2, 2]$), but the constant in $int sin(x) dx = -cos(x) + C$ can be any real number whatsoever, even one far outside this range, say assign $C := mbox{Graham's number}$. Thus the integrals with variable bounds are not exhaustive of the full family of antiderivatives, which is what is represented by the indefinite integral, and if we are really pedantic should be truly written as the set it is: in "reality", not in the informalities of school calculus,



$$int f(x) dx = left{ F(x) + C : C in mathbb{R} right}$$



where $F'(x) = f(x)$. Actually, if we want to be really correct - and in fact this is important but only in cases where the definite integral technically doesn't make sense as evaluable between arbitrary pairs of points - we should take



$$int f(x) dx = left{ F(x) in mathbb{R}^mathbb{R} : mbox{$F'$ exists and $F'(x) = f(x)$} right}$$.






share|cite|improve this answer























  • +1 I thought about going to this level of sophistication in my much shorter answer but decided the OP needed just the central points I made. Now they can see both views.
    – Ethan Bolker
    3 hours ago










  • Very pedantic answer that is 100% valid and makes a distinction I had never thought about. +1 from me too. Heck, I'd give it a +5 if it was possible/allowed.
    – JonathanZ
    3 hours ago












  • I'm having a hard time understanding this, as I'm new to Calculus, but it's ok.
    – clabe45
    3 mins ago



















4














That fixed but arbitrary lower limit is essentially the "arbitrary constant" you use when finding antiderivatives, because
$$
g(x) = int_a^x f(t), dt =
int_b^x f(t), dt +
int_a^b f(t), dt .
$$

The second term in that sum is just a number.



The fundamental theorem of calculus says that both integrals with upper limit $x$ have derivative $f$ with respect to $x$.



The "indefinite integral" with no limits specified refers to the whole set of antiderivatives, not any particular one of them.






share|cite|improve this answer





















  • I'm almost following what you're saying, but not quite. I understand that both integrals with upper limit $x$ have derivative $f$ with respect to $x$, but how does that make the constant integral the arbitrary constant?
    – clabe45
    1 hour ago











Your Answer





StackExchange.ifUsing("editor", function () {
return StackExchange.using("mathjaxEditing", function () {
StackExchange.MarkdownEditor.creationCallbacks.add(function (editor, postfix) {
StackExchange.mathjaxEditing.prepareWmdForMathJax(editor, postfix, [["$", "$"], ["\\(","\\)"]]);
});
});
}, "mathjax-editing");

StackExchange.ready(function() {
var channelOptions = {
tags: "".split(" "),
id: "69"
};
initTagRenderer("".split(" "), "".split(" "), channelOptions);

StackExchange.using("externalEditor", function() {
// Have to fire editor after snippets, if snippets enabled
if (StackExchange.settings.snippets.snippetsEnabled) {
StackExchange.using("snippets", function() {
createEditor();
});
}
else {
createEditor();
}
});

function createEditor() {
StackExchange.prepareEditor({
heartbeatType: 'answer',
autoActivateHeartbeat: false,
convertImagesToLinks: true,
noModals: true,
showLowRepImageUploadWarning: true,
reputationToPostImages: 10,
bindNavPrevention: true,
postfix: "",
imageUploader: {
brandingHtml: "Powered by u003ca class="icon-imgur-white" href="https://imgur.com/"u003eu003c/au003e",
contentPolicyHtml: "User contributions licensed under u003ca href="https://creativecommons.org/licenses/by-sa/3.0/"u003ecc by-sa 3.0 with attribution requiredu003c/au003e u003ca href="https://stackoverflow.com/legal/content-policy"u003e(content policy)u003c/au003e",
allowUrls: true
},
noCode: true, onDemand: true,
discardSelector: ".discard-answer"
,immediatelyShowMarkdownHelp:true
});


}
});














draft saved

draft discarded


















StackExchange.ready(
function () {
StackExchange.openid.initPostLogin('.new-post-login', 'https%3a%2f%2fmath.stackexchange.com%2fquestions%2f3056882%2fcan-an-indefinite-integral-be-expressed-as-a-definite-integral-with-variable-bou%23new-answer', 'question_page');
}
);

Post as a guest















Required, but never shown

























2 Answers
2






active

oldest

votes








2 Answers
2






active

oldest

votes









active

oldest

votes






active

oldest

votes









5














No, not always.



In fact, the definite integral with variable bounds can be and is strictly more restrictive than the indefinite integral, in the most general case where both can be taken as sensibly existing to their fullest extent that we can say one defines the same kind of function as the other.



While it is true that if we have two functions $F_1$ and $F_2$ defined by definite integrals of the same function with different bounds, i.e.



$$F_1(x) := int_{a_1}^{x} f(t) dt mathrm{and} F_2(x) := int_{a_2}^{x} f(t) dt$$



with $a_1 ne a_2$, then we will have that $F_1 - F_2$ is a constant function, the range of possible such constants may be limited. A simple counterexample is to take $f(x) := sin(x)$. Now the integral from any lower bound $a$ to $x$ is just $int_a^x sin(t) dt = cos(a) - cos(x)$, but here $cos(a)$ - the "constant" of our integration - can only range in $[-1, 1]$ (and the difference of any two such "constants" only within $[-2, 2]$), but the constant in $int sin(x) dx = -cos(x) + C$ can be any real number whatsoever, even one far outside this range, say assign $C := mbox{Graham's number}$. Thus the integrals with variable bounds are not exhaustive of the full family of antiderivatives, which is what is represented by the indefinite integral, and if we are really pedantic should be truly written as the set it is: in "reality", not in the informalities of school calculus,



$$int f(x) dx = left{ F(x) + C : C in mathbb{R} right}$$



where $F'(x) = f(x)$. Actually, if we want to be really correct - and in fact this is important but only in cases where the definite integral technically doesn't make sense as evaluable between arbitrary pairs of points - we should take



$$int f(x) dx = left{ F(x) in mathbb{R}^mathbb{R} : mbox{$F'$ exists and $F'(x) = f(x)$} right}$$.






share|cite|improve this answer























  • +1 I thought about going to this level of sophistication in my much shorter answer but decided the OP needed just the central points I made. Now they can see both views.
    – Ethan Bolker
    3 hours ago










  • Very pedantic answer that is 100% valid and makes a distinction I had never thought about. +1 from me too. Heck, I'd give it a +5 if it was possible/allowed.
    – JonathanZ
    3 hours ago












  • I'm having a hard time understanding this, as I'm new to Calculus, but it's ok.
    – clabe45
    3 mins ago
















5














No, not always.



In fact, the definite integral with variable bounds can be and is strictly more restrictive than the indefinite integral, in the most general case where both can be taken as sensibly existing to their fullest extent that we can say one defines the same kind of function as the other.



While it is true that if we have two functions $F_1$ and $F_2$ defined by definite integrals of the same function with different bounds, i.e.



$$F_1(x) := int_{a_1}^{x} f(t) dt mathrm{and} F_2(x) := int_{a_2}^{x} f(t) dt$$



with $a_1 ne a_2$, then we will have that $F_1 - F_2$ is a constant function, the range of possible such constants may be limited. A simple counterexample is to take $f(x) := sin(x)$. Now the integral from any lower bound $a$ to $x$ is just $int_a^x sin(t) dt = cos(a) - cos(x)$, but here $cos(a)$ - the "constant" of our integration - can only range in $[-1, 1]$ (and the difference of any two such "constants" only within $[-2, 2]$), but the constant in $int sin(x) dx = -cos(x) + C$ can be any real number whatsoever, even one far outside this range, say assign $C := mbox{Graham's number}$. Thus the integrals with variable bounds are not exhaustive of the full family of antiderivatives, which is what is represented by the indefinite integral, and if we are really pedantic should be truly written as the set it is: in "reality", not in the informalities of school calculus,



$$int f(x) dx = left{ F(x) + C : C in mathbb{R} right}$$



where $F'(x) = f(x)$. Actually, if we want to be really correct - and in fact this is important but only in cases where the definite integral technically doesn't make sense as evaluable between arbitrary pairs of points - we should take



$$int f(x) dx = left{ F(x) in mathbb{R}^mathbb{R} : mbox{$F'$ exists and $F'(x) = f(x)$} right}$$.






share|cite|improve this answer























  • +1 I thought about going to this level of sophistication in my much shorter answer but decided the OP needed just the central points I made. Now they can see both views.
    – Ethan Bolker
    3 hours ago










  • Very pedantic answer that is 100% valid and makes a distinction I had never thought about. +1 from me too. Heck, I'd give it a +5 if it was possible/allowed.
    – JonathanZ
    3 hours ago












  • I'm having a hard time understanding this, as I'm new to Calculus, but it's ok.
    – clabe45
    3 mins ago














5












5








5






No, not always.



In fact, the definite integral with variable bounds can be and is strictly more restrictive than the indefinite integral, in the most general case where both can be taken as sensibly existing to their fullest extent that we can say one defines the same kind of function as the other.



While it is true that if we have two functions $F_1$ and $F_2$ defined by definite integrals of the same function with different bounds, i.e.



$$F_1(x) := int_{a_1}^{x} f(t) dt mathrm{and} F_2(x) := int_{a_2}^{x} f(t) dt$$



with $a_1 ne a_2$, then we will have that $F_1 - F_2$ is a constant function, the range of possible such constants may be limited. A simple counterexample is to take $f(x) := sin(x)$. Now the integral from any lower bound $a$ to $x$ is just $int_a^x sin(t) dt = cos(a) - cos(x)$, but here $cos(a)$ - the "constant" of our integration - can only range in $[-1, 1]$ (and the difference of any two such "constants" only within $[-2, 2]$), but the constant in $int sin(x) dx = -cos(x) + C$ can be any real number whatsoever, even one far outside this range, say assign $C := mbox{Graham's number}$. Thus the integrals with variable bounds are not exhaustive of the full family of antiderivatives, which is what is represented by the indefinite integral, and if we are really pedantic should be truly written as the set it is: in "reality", not in the informalities of school calculus,



$$int f(x) dx = left{ F(x) + C : C in mathbb{R} right}$$



where $F'(x) = f(x)$. Actually, if we want to be really correct - and in fact this is important but only in cases where the definite integral technically doesn't make sense as evaluable between arbitrary pairs of points - we should take



$$int f(x) dx = left{ F(x) in mathbb{R}^mathbb{R} : mbox{$F'$ exists and $F'(x) = f(x)$} right}$$.






share|cite|improve this answer














No, not always.



In fact, the definite integral with variable bounds can be and is strictly more restrictive than the indefinite integral, in the most general case where both can be taken as sensibly existing to their fullest extent that we can say one defines the same kind of function as the other.



While it is true that if we have two functions $F_1$ and $F_2$ defined by definite integrals of the same function with different bounds, i.e.



$$F_1(x) := int_{a_1}^{x} f(t) dt mathrm{and} F_2(x) := int_{a_2}^{x} f(t) dt$$



with $a_1 ne a_2$, then we will have that $F_1 - F_2$ is a constant function, the range of possible such constants may be limited. A simple counterexample is to take $f(x) := sin(x)$. Now the integral from any lower bound $a$ to $x$ is just $int_a^x sin(t) dt = cos(a) - cos(x)$, but here $cos(a)$ - the "constant" of our integration - can only range in $[-1, 1]$ (and the difference of any two such "constants" only within $[-2, 2]$), but the constant in $int sin(x) dx = -cos(x) + C$ can be any real number whatsoever, even one far outside this range, say assign $C := mbox{Graham's number}$. Thus the integrals with variable bounds are not exhaustive of the full family of antiderivatives, which is what is represented by the indefinite integral, and if we are really pedantic should be truly written as the set it is: in "reality", not in the informalities of school calculus,



$$int f(x) dx = left{ F(x) + C : C in mathbb{R} right}$$



where $F'(x) = f(x)$. Actually, if we want to be really correct - and in fact this is important but only in cases where the definite integral technically doesn't make sense as evaluable between arbitrary pairs of points - we should take



$$int f(x) dx = left{ F(x) in mathbb{R}^mathbb{R} : mbox{$F'$ exists and $F'(x) = f(x)$} right}$$.







share|cite|improve this answer














share|cite|improve this answer



share|cite|improve this answer








edited 3 hours ago

























answered 3 hours ago









The_Sympathizer

7,3802244




7,3802244












  • +1 I thought about going to this level of sophistication in my much shorter answer but decided the OP needed just the central points I made. Now they can see both views.
    – Ethan Bolker
    3 hours ago










  • Very pedantic answer that is 100% valid and makes a distinction I had never thought about. +1 from me too. Heck, I'd give it a +5 if it was possible/allowed.
    – JonathanZ
    3 hours ago












  • I'm having a hard time understanding this, as I'm new to Calculus, but it's ok.
    – clabe45
    3 mins ago


















  • +1 I thought about going to this level of sophistication in my much shorter answer but decided the OP needed just the central points I made. Now they can see both views.
    – Ethan Bolker
    3 hours ago










  • Very pedantic answer that is 100% valid and makes a distinction I had never thought about. +1 from me too. Heck, I'd give it a +5 if it was possible/allowed.
    – JonathanZ
    3 hours ago












  • I'm having a hard time understanding this, as I'm new to Calculus, but it's ok.
    – clabe45
    3 mins ago
















+1 I thought about going to this level of sophistication in my much shorter answer but decided the OP needed just the central points I made. Now they can see both views.
– Ethan Bolker
3 hours ago




+1 I thought about going to this level of sophistication in my much shorter answer but decided the OP needed just the central points I made. Now they can see both views.
– Ethan Bolker
3 hours ago












Very pedantic answer that is 100% valid and makes a distinction I had never thought about. +1 from me too. Heck, I'd give it a +5 if it was possible/allowed.
– JonathanZ
3 hours ago






Very pedantic answer that is 100% valid and makes a distinction I had never thought about. +1 from me too. Heck, I'd give it a +5 if it was possible/allowed.
– JonathanZ
3 hours ago














I'm having a hard time understanding this, as I'm new to Calculus, but it's ok.
– clabe45
3 mins ago




I'm having a hard time understanding this, as I'm new to Calculus, but it's ok.
– clabe45
3 mins ago











4














That fixed but arbitrary lower limit is essentially the "arbitrary constant" you use when finding antiderivatives, because
$$
g(x) = int_a^x f(t), dt =
int_b^x f(t), dt +
int_a^b f(t), dt .
$$

The second term in that sum is just a number.



The fundamental theorem of calculus says that both integrals with upper limit $x$ have derivative $f$ with respect to $x$.



The "indefinite integral" with no limits specified refers to the whole set of antiderivatives, not any particular one of them.






share|cite|improve this answer





















  • I'm almost following what you're saying, but not quite. I understand that both integrals with upper limit $x$ have derivative $f$ with respect to $x$, but how does that make the constant integral the arbitrary constant?
    – clabe45
    1 hour ago
















4














That fixed but arbitrary lower limit is essentially the "arbitrary constant" you use when finding antiderivatives, because
$$
g(x) = int_a^x f(t), dt =
int_b^x f(t), dt +
int_a^b f(t), dt .
$$

The second term in that sum is just a number.



The fundamental theorem of calculus says that both integrals with upper limit $x$ have derivative $f$ with respect to $x$.



The "indefinite integral" with no limits specified refers to the whole set of antiderivatives, not any particular one of them.






share|cite|improve this answer





















  • I'm almost following what you're saying, but not quite. I understand that both integrals with upper limit $x$ have derivative $f$ with respect to $x$, but how does that make the constant integral the arbitrary constant?
    – clabe45
    1 hour ago














4












4








4






That fixed but arbitrary lower limit is essentially the "arbitrary constant" you use when finding antiderivatives, because
$$
g(x) = int_a^x f(t), dt =
int_b^x f(t), dt +
int_a^b f(t), dt .
$$

The second term in that sum is just a number.



The fundamental theorem of calculus says that both integrals with upper limit $x$ have derivative $f$ with respect to $x$.



The "indefinite integral" with no limits specified refers to the whole set of antiderivatives, not any particular one of them.






share|cite|improve this answer












That fixed but arbitrary lower limit is essentially the "arbitrary constant" you use when finding antiderivatives, because
$$
g(x) = int_a^x f(t), dt =
int_b^x f(t), dt +
int_a^b f(t), dt .
$$

The second term in that sum is just a number.



The fundamental theorem of calculus says that both integrals with upper limit $x$ have derivative $f$ with respect to $x$.



The "indefinite integral" with no limits specified refers to the whole set of antiderivatives, not any particular one of them.







share|cite|improve this answer












share|cite|improve this answer



share|cite|improve this answer










answered 3 hours ago









Ethan Bolker

41.2k547108




41.2k547108












  • I'm almost following what you're saying, but not quite. I understand that both integrals with upper limit $x$ have derivative $f$ with respect to $x$, but how does that make the constant integral the arbitrary constant?
    – clabe45
    1 hour ago


















  • I'm almost following what you're saying, but not quite. I understand that both integrals with upper limit $x$ have derivative $f$ with respect to $x$, but how does that make the constant integral the arbitrary constant?
    – clabe45
    1 hour ago
















I'm almost following what you're saying, but not quite. I understand that both integrals with upper limit $x$ have derivative $f$ with respect to $x$, but how does that make the constant integral the arbitrary constant?
– clabe45
1 hour ago




I'm almost following what you're saying, but not quite. I understand that both integrals with upper limit $x$ have derivative $f$ with respect to $x$, but how does that make the constant integral the arbitrary constant?
– clabe45
1 hour ago


















draft saved

draft discarded




















































Thanks for contributing an answer to Mathematics Stack Exchange!


  • Please be sure to answer the question. Provide details and share your research!

But avoid



  • Asking for help, clarification, or responding to other answers.

  • Making statements based on opinion; back them up with references or personal experience.


Use MathJax to format equations. MathJax reference.


To learn more, see our tips on writing great answers.





Some of your past answers have not been well-received, and you're in danger of being blocked from answering.


Please pay close attention to the following guidance:


  • Please be sure to answer the question. Provide details and share your research!

But avoid



  • Asking for help, clarification, or responding to other answers.

  • Making statements based on opinion; back them up with references or personal experience.


To learn more, see our tips on writing great answers.




draft saved


draft discarded














StackExchange.ready(
function () {
StackExchange.openid.initPostLogin('.new-post-login', 'https%3a%2f%2fmath.stackexchange.com%2fquestions%2f3056882%2fcan-an-indefinite-integral-be-expressed-as-a-definite-integral-with-variable-bou%23new-answer', 'question_page');
}
);

Post as a guest















Required, but never shown





















































Required, but never shown














Required, but never shown












Required, but never shown







Required, but never shown

































Required, but never shown














Required, but never shown












Required, but never shown







Required, but never shown







Popular posts from this blog

Plaza Victoria

In PowerPoint, is there a keyboard shortcut for bulleted / numbered list?

How to put 3 figures in Latex with 2 figures side by side and 1 below these side by side images but in...